Как решить «ЭМ волновое уравнение» для поля равномерно движущегося заряда?

Можно ли показать, что поле равномерно движущегося заряда, соответствующее закону Био-Савара, определяется выражением...

Е ( р , т ) "=" к д ( 1 в 2 / с 2 ( 1 в 2 грех 2 θ / с 2 ) 3 / 2 ) ( р ^ γ 2 ( Икс в т ) 2 + у 2 + г 2 )

... удовлетворяет приведенному ниже уравнению (вдали от самого заряда)?

с 2 2 Е "=" 2 т 2 Е

Редактировать:

Я знаю , что равномерное движение не создает волн. Я также знаю , что это уравнение справедливо только для пустого пространства вдали от зарядов и токов.

Но каким-то образом это уравнение все же должно выполняться для электрического поля, «тянущегося» по инерции за зарядом, движущимся с постоянной скоростью.

Мне очень трудно понять эту мысль. Любые ссылки будут приветствоваться. Я пытался решить напрямую, но не удалось.

Вы должны отредактировать свой пост, чтобы было ясно, что вы знаете, что равномерно движущийся заряд не генерирует волну, но поля, по-видимому, должны удовлетворять этому уравнению. Я не знаю ответа (вопрос за голосование), но я отмечаю, что в вашем волновом уравнении нет исходного члена ... возможно, это имеет значение.
Один из подходов к этому состоял бы в том, чтобы сначала перейти к системе покоя заряда, в которой электрическое поле особенно просто, а именно Е "=" д 4 π ϵ 0 р 2 , где нетрудно показать (поскольку нет производных по времени), затем перейти к системе отсчета, в которой движется заряд, и отметить, что 2 т 2 + 1 с 2 2 является инвариантным, см., например, Purcell, Electricity and Magnetism .
Привет, @jim, правильно ли говорить, что так называемое «уравнение электромагнитной волны» имеет явно неволновые решения (например, поле, движущееся по инерции)? Почему оно все-таки называется волновым уравнением?
@VitalyKorzhik Я думаю, что, поскольку уравнение ЭМ может принимать волнообразные решения, оно называется волновым уравнением ЭМ . Когда вы смотрите на уравнения, не зависящие от времени, не делается особого различия. Кроме того, у вас могут быть решения, зависящие от времени, которые не обязательно должны быть синусоидальными, которые люди обычно рассматривают как волну.
@jim Есть ли какая-то очевидная причина, по которой функция поля не удовлетворит в 2 2 Е "=" 2 т 2 Е в вместо с )? Потому что это не может удовлетворить обоих одновременно. Вероятно, это как-то связано с преобразованием Лоренца...
Да, преобразование Лоренца означает, что только когда в "=" с является инвариантом волнового уравнения. Другой способ думать об этом - для в < с можно было бы перейти к кадру, где в "=" 0 это означает, что вы можете увеличить кадр, где электромагнитные волны будут статичными. Следующий вопрос и ответ могут представлять интерес: physics.stackexchange.com/questions/248499/…

Ответы (1)

Люди не понимают вашего вопроса. Я думаю, вы хотите, чтобы кто-то явно проверил, что поля, создаваемые в вашем частном случае, подчиняются (или не подчиняются) общепринятому волновому уравнению. Ведь создаваемое поле не похоже на волну. Общее решение волнового уравнения для возмущения, бегущего в Икс направление Е ( р , т ) "=" Е 0 ( Икс с т , у , г ) . Так что я полагаю

Е ( р , т ) "=" 1 4 π ϵ 0 д [ ( Икс с т ) 2 + у 2 + г 2 ] р ^
(где р ^ точки от заряда в любой момент) является решением волнового уравнения.

Очевидно, это хорошо только для нерелятивистских скоростей. И мои обозначения для единичного вектора не очень хороши ... Я надеюсь, что это имеет смысл.

Комментарий

Я вижу, что вы отредактировали свой вопрос, включив в него явное выражение релятивистского выражения для поля. Я думаю, ясно, что то, что я сделал для нерелятивистского случая, работает так же хорошо и для релятивистского случая.

Я удалил этот комментарий и добавил его в свой ответ.
Я добавил зависимость от времени явно в соответствии с вашими предложениями, надеюсь, это выглядит правильно. Я думаю, релятивистскую часть можно игнорировать для простоты @Mathaholic
Это почти правильно ... вы не определяете р ^ , и я тоже. При правильном определении р ^ , я думаю то что вы написали явно решение волнового уравнения!
Проблема в том, что у меня есть в вместо с в моем уравнении, поэтому я не понимаю, как это решает уравнение Максвелла с с в этом. Можно немного уточнить?
Ага. Я знал, что это выглядело слишком просто. :)